IFRAME SYNC
IFRAME SYNC
IFRAME SYNC
IFRAME SYNC

Weird limit: $\lim_{n\to\infty} \,\sum_{k=1}^n\left(\frac{k}{n^2}\right)^{\frac{k}{n^2}+1}$ https://ift.tt/eA8V8J

I came across the following problem recently:

Evaluate $$\lim_{n\to\infty} \,\sum_{k=1}^n\left(\frac{k}{n^2}\right)^{\frac{k}{n^2}+1}$$

I tried to rewrite the inner sum as a Riemann sum hoping that the limit would become a definite integral, but no gain. Please help!



from Hot Weekly Questions - Mathematics Stack Exchange
Alan

Post a Comment

[blogger]

Contact Form

Name

Email *

Message *

copyrighted to mathematicianadda.com. Powered by Blogger.
Javascript DisablePlease Enable Javascript To See All Widget

Blog Archive